Un producto tensorial de dos partículas de spin-1

Estoy bastante confundido, y esperaba que alguien pudiera ayudarme a resolver este problema (probablemente bastante elemental). Tengo dos partículas con espín 1, cuyo estado describo por metro S y metro I respectivamente. Ambos pueden tomar los valores -1, 0 y 1.

Ahora, lo que quiero hacer es calcular un producto tensorial. Ahora, aquí es donde podría estar equivocándome, pero por supuesto tengo que elegir una base. ¿Sería posible decir que | metro s = 1 = ( 1 0 0 ) , | metro s = 0 = ( 0 1 0 ) , | metro s = 1 = ( 0 0 1 ) , | metro I = 1 = ( 1 0 0 ) , | metro I = 0 = ( 0 1 0 ) , | metro I = 1 = ( 0 0 1 )

Supongo que aquí podría ser donde las cosas empiezan a salir mal, ya que tal vez no pueda elegir los mismos vectores base para dos partículas. De todos modos, SI lo anterior es cierto, me gustaría, por ejemplo, calcular

1 2 ( | metro s = 0 i | metro s = 1 ) | metro I = 1

Ahora, tal como lo veo, esto sería simplemente un vector de 9x1 igual a

( 0 0 0 0 0 1 2 0 0 i 2 )

Sin embargo, no estoy seguro si lo que estoy escribiendo aquí es correcto. En la siguiente parte de mis cálculos, introduzco un término hamiltoniano, que hace que el espín de la primera partícula evolucione dependiendo del espín de la segunda partícula, y al resolver los términos parece que las cosas salen terriblemente mal. Como el hamiltoniano es simplemente una constante multiplicada por el producto tensorial de las dos matrices de espín z pauli para una partícula de espín 1, no hay mucho que pueda salir mal allí, así que pensé que el error debe estar aquí en alguna parte.

Hm, comentario rápido, acabo de notar algo que hice mal en la parte donde no podía salir mucho mal. Tal vez pueda borrar la publicación.
Aún mejor: publica la solución.

Respuestas (1)

Las dos partículas metro s y metro I vive en diferentes espacios vectoriales, por lo que en realidad no está eligiendo los mismos vectores base (porque los vectores base de las diferentes partículas pertenecen a dos espacios vectoriales separados).

En segundo lugar, el producto tensorial entre los vectores base de los dos espacios vectoriales diferentes formará los vectores base de un nuevo 3 × 3 = 9 espacio vectorial dimensional. Por ejemplo:

| metro s = 1 | metro I = 0 = ( 0 1 0 0 0 0 0 0 0 )
Así que la ecuación que has escrito es correcta.

Recomiendo encarecidamente este breve conjunto de notas para tener una mejor idea de la maquinaria completa.

Editar en respuesta al comentario:

Necesitamos usar el producto tensorial para sumar momentos angulares. Para simplificar, considere un sistema con dos partículas, y el giro de ambas partículas es 1 / 2 . Entonces, estas partículas se pueden combinar de la siguiente manera:

↑↑ , ↑↓ , ↓↑ , ↓↓
Esto significa que el espacio de Hilbert de dos partículas (es decir, el espacio de Hilbert correspondiente al sistema) está dividido por cuatro vectores base:
| s 1 , metro 1 ; s 2 , metro 2 | s 1 , metro 1 | s 2 , metro 2
Posteriormente, como he esbozado en esta respuesta , descomponemos este producto tensorial para determinar cuáles son los valores propios posibles para la magnitud y z -componente del sistema total (es decir, el sistema formado por los dos spin- 1 / 2 partículas) puede ser.

Estoy seguro de que me estoy perdiendo algo, pero ¿no estamos agregando estados propios de momento angular? Entonces, ¿no debería ser la respuesta una combinación lineal de estados ponderados por sus coeficientes de Clebsch Gordan?
@JeffDror He actualizado mi mensaje. Espero que esto responda a su pregunta (más o menos).
@JeffDror también, el OP no pide directamente agregar el momento angular de las dos partículas.
Muchas gracias por la ayuda, y también especialmente por esas notas, ¡muy útiles!
@ usuario129412 no hay problema. El producto tensorial siempre me pareció un poco mágico, hasta que leí esas notas. Así que me alegra saber que también los encontró útiles.